高等代数 - Eufisky - The lost book

常用技巧

爱因斯坦名言“一个人的价值,应该看他贡献什么,而不应当看他取得什么。”

1、矩阵的 Frobenius 范数及其求偏导法则

2、关于数学优化算法的权威期刊都有哪些?

1. SIAM Journal on Optimization
2. Mathematical Programming
3. Mathematical Programming Computation
4. Computational Optimization and Applications
5. Optimization Methods and Software
 
作者:Stiefel
链接:https://www.zhihu.com/question/39194485/answer/129350231
来源:知乎
著作权归作者所有。商业转载请联系作者获得授权,非商业转载请注明出处。

 

 

Hurwitz 平方和定理

Hurwitz 平方和定理是有限群表示论的一个精彩应用,本文是若干年前读书时的笔记。
 
Hurwitz 平方和定理
我们都熟悉复数的乘法:如果 z_1=x_1+y_1i,z_2=x_2+y_2i 是两个复数,则 |z_1z_2|=|z_1|\cdot|z_2|,也就是
(x_1^2+y_1^2)(x_2^2+y_2^2)=(x_1x_2-y_1y_2)^2+(x_1y_2+x_2y_1)^2.
1748 年 Euler 发现了如下的 4 平方和等式:
(x_1^2+x_2^2+x_3^2+x_4^2)(y_1^2+y_2^2+y_3^2+y_4^2)=z_1^2+z_2^2+z_3^2+z_4^2.
其中
\begin{align*}&z_1=x_1y_1-x_2y_2-x_3y_3-x_4y_4,\\&z_2=x_1y_2+x_2y_1+x_3y_4-x_4y_3,\\&z_3=x_1y_3+x_3y_1-x_2y_4+x_4y_2,\\&z_4=x_1y_4+x_4y_1+x_2y_3-x_3y_2.\end{align*}
4 平方和等式说的是在 Hamilton 四元数体中范数仍然是乘性的。1848 年 Caley 发现了八元数,从而导出了类似的 8 平方和等式,当然具体写出来会很复杂,这里就按下不表了。
 
一般地,如果能在 n 维欧式空间 \mathbb{R}^n 上定义向量之间的乘法:
\mathbb{R^n}\times\mathbb{R^n}\rightarrow\mathbb{R^n}:(v,w)\rightarrow v\times w
使得 v\times w 对 v,w 都是线性的,而且乘积的范数等于范数的乘积:|v\times w|=|v|\cdot |w|(这里 |\cdot| 是通常的欧式范数),则我们就得到了一个 n 平方和等式。
 
在接下来的 50 年里,人们一直致力于寻找可能的 16 平方和等式,但是都失败了,于是开始怀疑是否没有这样的等式成立。终于在 1898 年 Hurwitz 证明了这样的结论:
 
Hurwitz 平方和定理:设 x=(x_1,\ldots,x_n), y=(y_1,\dots,y_n) 为 \mathbb{R}^n 中的向量。如果存在关于 x,y 的双线性函数 z_1(x,y),\ldots,z_n(x,y) 使得等式
(x_1^2+\cdots+x_n^2)(y_1^2+\cdots+y_n^2)=z_1^2+\cdots+z_n^2
恒成立, 那么 n=1,2,4,8。
正如前面说过的,Huiwitz 平方和定理说的是在实数域 \mathbb{R},复数域 \mathbb{C},四元数 \mathbb{H} 和八元数 \mathbb{O} 中,元素的(欧式)范数和向量的乘法是相容的,而在其它维数的 \mathbb{R}^n 上是不可能定义与欧式范数相容的向量乘法的。
 
Hurwitz 本人的证明是纯线性代数的,线性代数的证明较为初等,不过步骤略长。1943 年 Eckmann 用有限群表示论的方法给了一个漂亮的证明,本文就来介绍这个证明。
 
将问题转化为矩阵方程
 
设 z=(z_1,\ldots,z_n),则 z 关于 y 是线性的,因此存在 n 阶矩阵 A 满足 z=yA,当然矩阵 A 和 x 有关。于是 Hurwitz 定理中的等式变成
(x_1^2+x_2^2+\cdots+x_n^2)yy'=yAA'y'.
由于 y 是不定元,因此
AA'=(x_1^2+\cdots+x_n^2)I_n.
进一步,由于 A 关于 x 也是线性的,因此设 A=A_1x_1+\cdots+A_nx_n,则
AA'=\sum_{i=1}^nA_iA_i'x_i^2+\sum_{1\leq i<j\leq n}(A_iA_j'+A_jA_i')x_ix_j.
从而我们得到一组矩阵方程
A_iA_i'=I_n,\quad A_iA_j'+A_jA_i'=0 \quad \text{for}\ i\ne j.
进一步可以把 A_n 归一化为单位矩阵:令 Q_i=A_iA_n^{-1},于是 Q_1,\ldots,Q_{n-1} 满足
Q_i'=-Q_i,\quad Q_i^2=-I_n,\quad Q_iQ_j=-Q_jQ_i\quad\text{for}\ i\ne j.
显然 n 必须是偶数(奇数阶反对称矩阵行列式都是 0),而 n=2 的时候结论是成立的,所以下面我们都假定 n>2,于是 n 的可能值为 4,6,8,\ldots
 
用群表示论的工具得出矛盾
 
考虑这样一个抽象群 G,它由元素 a,g_1,\ldots,g_{n-1} 生成,且
a^2=1,\quad g_i^2=a,\quad g_ig_j=ag_jg_i\ \text{when}\ i\ne j.
这个群的结构很好分析:
 
|G|=2^n,每个元素形如 a^{e_0}g_1^{e_1}\cdots g_{n-1}^{e_{n-1}},其中 e_i\in\{0,1\}。
G 的中心 Z(G)=\{1,a,g_1g_2\cdots g_{n-1},ag_1g_2\cdots g_{n-1}\}。
G 的换位子群 [G,G]=\{1,a\},从而 G 有 2^{n-1} 个线性表示。
G 的任何非平凡共轭类都只有两个元素 \{g,ag\},从而 G 有 2^{n-1}+2 个共轭类,其不可约复表示的个数也是 2^{n-1}+2。
于是我们知道 G 有 2^{n-1} 个一次表示,还有 2 个次数大于 1 的表示,设它俩的次数分别是 f_1,f_2,根据不可约表示次数的平方和等于 G 的阶,得到方程
f_1^2+f_2^2 =2^{n-1}.
再利用不可约表示的次数整除 G 的阶,知道 f_1 和 f_2 都是 2 的幂,这只有一种可能,就是
f_1=f_2=2^{\frac{n}{2}-1}.
 
现在 Hurwitz 矩阵方程给出了 G 的一个 n 维表示,这个表示可以分解为若干不可约表示的直和,我们断言其中不含有一次表示,从而只能是若干个 2^{\frac{n}{2}-1} 次表示的直和:这是因为元素 a 在这个表示下是 n 阶矩阵 -I_n,从而其在任何不变子空间上的作用都是乘以 -1。但是任何一次表示都把 a\in [G,G] 映射为 1,矛盾!
 
于是 2^{\frac{n}{2}-1}\big| n,设 n=2^r\cdot s,其中 s 为奇数,则 \frac{n}{2}-1\leq r,从而
2^r\leq n\leq 2r+2.
注意 n 是偶数,所以只能是 n=4,6,8,这就完成了 Hurwitz 定理的证明。

十个利用矩阵解决的经典题目

来源:http://blog.csdn.net/fun_zero/article/details/48685661

经典题目1 给定n个点,m个操作,构造O(m+n)的算法输出m个操作后各点的位置。操作有平移、缩放、翻转和旋转

    这里的操作是对所有点同时进行的。其中翻转是以坐标轴为对称轴进行翻转(两种情况),旋转则以原点为中心。如果对每个点分别进行模拟,那么m个操作总共耗时O(mn)。利用矩阵乘法可以在O(m)的时间里把所有操作合并为一个矩阵,然后每个点与该矩阵相乘即可直接得出最终该点的位置,总共耗时O(m+n)。假设初始时某个点的坐标为x和y,下面5个矩阵可以分别对其进行平移、旋转、翻转和旋转操作。预先把所有m个操作所对应的矩阵全部乘起来,再乘以(x,y,1),即可一步得出最终点的位置。

     

经典题目2 给定矩阵A,请快速计算出A^n(n个A相乘)的结果,输出的每个数都mod p。

    由于矩阵乘法具有结合律,因此A^4 = A * A * A * A = (A*A) * (A*A) = A^2 * A^2。我们可以得到这样的结论:当n为偶数时,A^n = A^(n/2) * A^(n/2);当n为奇数时,A^n = A^(n/2) * A^(n/2) * A (其中n/2取整)。这就告诉我们,计算A^n也可以使用二分快速求幂的方法。例如,为了算出A^25的值,我们只需要递归地计算出A^12、A^6、A^3的值即可。根据这里的一些结果,我们可以在计算过程中不断取模,避免高精度运算。

经典题目POJ3233 (感谢rmq)
    
题目大意:给定矩阵A,求A + A^2 + A^3 + ... + A^k的结果(两个矩阵相加就是对应位置分别相加)。输出的数据mod mk<=10^9
    这道题两次二分,相当经典。首先我们知道,A^i可以二分求出。然后我们需要对整个题目的数据规模k进行二分。比如,当k=6时,有:
    A + A^2 + A^3 + A^4 + A^5 + A^6 =(A + A^2 + A^3) + A^3*(A + A^2 + A^3)

    应用这个式子后,规模k减小了一半。我们二分求出A^3后再递归地计算A + A^2 + A^3,即可得到原问题的答案。

POJ3233题解: Matrix Power Series


经典题目4 VOJ1049
    题目大意:顺次给出m个置换,反复使用这m个置换对初始序列进行操作,问k次置换后的序列。m<=10, k<2^31。
    首先将这m个置换“合并”起来(算出这m个置换的乘积),然后接下来我们需要执行这个置换k/m次(取整,若有余数则剩下几步模拟即可)。注意任意一个置换都可以表示成矩阵的形式。例如,将1 2 3 4置换为3 1 2 4,相当于下面的矩阵乘法:
     
    置换k/m次就相当于在前面乘以k/m个这样的矩阵。我们可以二分计算出该矩阵的k/m次方,再乘以初始序列即可。做出来了别忙着高兴,得意之时就是你灭亡之日,别忘了最后可能还有几个置换需要模拟。

经典题目5 《算法艺术与信息学竞赛》207页(2.1代数方法和模型,[例题5]细菌,版次不同可能页码有偏差)
    大家自己去看看吧,书上讲得很详细。解题方法和上一题类似,都是用矩阵来表示操作,然后二分求最终状态。

经典题目6 给定n和p,求第n个Fibonacci数mod p的值,n不超过2^31
    根据前面的一些思路,现在我们需要构造一个2 x 2的矩阵,使得它乘以(a,b)得到的结果是(b,a+b)。每多乘一次这个矩阵,这两个数就会多迭代一次。那么,我们把这个2 x 2的矩阵自乘n次,再乘以(0,1)就可以得到第n个Fibonacci数了。不用多想,这个2 x 2的矩阵很容易构造出来:
     

经典题目7 VOJ1067
    我们可以用上面的方法二分求出任何一个线性递推式的第n项,其对应矩阵的构造方法为:在右上角的(n-1)*(n-1)的小矩阵中的主对角线上填1,矩阵第n行填对应的系数,其它地方都填0。例如,我们可以用下面的矩阵乘法来二分计算f(n) = 4f(n-1) - 3f(n-2) + 2f(n-4)的第k项:
     
    利用矩阵乘法求解线性递推关系的题目我能编出一卡车来。这里给出的例题是系数全为1的情况。

 

经典题目8 给定一个有向图,问从A点恰好走k步(允许重复经过边)到达B点的方案数mod p的值

    把给定的图转为邻接矩阵,即A(i,j)=1当且仅当存在一条边i->j。令C=A*A,那么C(i,j)=ΣA(i,k)*A(k,j),实际上就等于从点i到点j恰好经过2条边的路径数(枚举k为中转点)。类似地,C*A的第i行第j列就表示从i到j经过3条边的路径数。同理,如果要求经过k步的路径数,我们只需要二分求出A^k即可。

例:HDU 2254   HDU 5302

 

经典题目9 用1 x 2的多米诺骨牌填满M x N的矩形有多少种方案,M<=5,N<2^31,输出答案mod p的结果
     
    我们以M=3为例进行讲解。假设我们把这个矩形横着放在电脑屏幕上,从右往左一列一列地进行填充。其中前n-2列已经填满了,第n-1列参差不齐。现在我们要做的事情是把第n-1列也填满,将状态转移到第n列上去。由于第n-1列的状态不一样(有8种不同的状态),因此我们需要分情况进行讨论。在图中,我把转移前8种不同的状态放在左边,转移后8种不同的状态放在右边,左边的某种状态可以转移到右边的某种状态就在它们之间连一根线。注意为了保证方案不重复,状态转移时我们不允许在第n-1列竖着放一个多米诺骨牌(例如左边第2种状态不能转移到右边第4种状态),否则这将与另一种转移前的状态重复。把这8种状态的转移关系画成一个有向图,那么问题就变成了这样:从状态111出发,恰好经过n步回到这个状态有多少种方案。比如,n=2时有3种方案,111->011->111、111->110->111和111->000->111,这与用多米诺骨牌覆盖3x2矩形的方案一一对应。这样这个题目就转化为了我们前面的例题8。
    后面我写了一份此题的源代码。你可以再次看到位运算的相关应用。

经典题目10 POJ2778
    题目大意是,检测所有可能的n位DNA串有多少个DNA串中不含有指定的病毒片段。合法的DNA只能由ACTG四个字符构成。题目将给出10个以内的病毒片段,每个片段长度不超过10。数据规模n<=2 000 000 000。
    下面的讲解中我们以ATC,AAA,GGC,CT这四个病毒片段为例,说明怎样像上面的题一样通过构图将问题转化为例题8。我们找出所有病毒片段的前缀,把n位DNA分为以下7类:以AT结尾、以AA结尾、以GG结尾、以?A结尾、以?G结尾、以?C结尾和以??结尾。其中问号表示“其它情况”,它可以是任一字母,只要这个字母不会让它所在的串成为某个病毒的前缀。显然,这些分类是全集的一个划分(交集为空,并集为全集)。现在,假如我们已经知道了长度为n-1的各类DNA中符合要求的DNA个数,我们需要求出长度为n时各类DNA的个数。我们可以根据各类型间的转移构造一个边上带权的有向图。例如,从AT不能转移到AA,从AT转移到??有4种方法(后面加任一字母),从?A转移到AA有1种方案(后面加个A),从?A转移到??有2种方案(后面加G或C),从GG到??有2种方案(后面加C将构成病毒片段,不合法,只能加A和T)等等。这个图的构造过程类似于用有限状态自动机做串匹配。然后,我们就把这个图转化成矩阵,让这个矩阵自乘n次即可。最后输出的是从??状态到所有其它状态的路径数总和。
    题目中的数据规模保证前缀数不超过100,一次矩阵乘法是三方的,一共要乘log(n)次。因此这题总的复杂度是100^3 * log(n),AC了。

问题征解1

1.(高等代数)证明:实对称矩阵${A_n} = \left[ {\begin{array}{*{20}{c}}{\frac{1}{1}}&{\frac{1}{2}}&{\frac{1}{3}}& \cdots &{\frac{1}{n}}\\{\frac{1}{2}}&{\frac{1}{2}}&{\frac{1}{3}}& \cdots &{\frac{1}{n}}\\{\frac{1}{3}}&{\frac{1}{3}}&{\frac{1}{3}}& \cdots &{\frac{1}{n}}\\\vdots & \vdots & \vdots &{}& \vdots \\{\frac{1}{n}}&{\frac{1}{n}}&{\frac{1}{n}}& \cdots &{\frac{1}{n}}\end{array}} \right]$的特征值都大于$0$,且小于等于$3+2\sqrt{2}$.


证.感谢Veer的提示(其实是cholesky分解),我已经证明出来了.
首先注意到\[A_n=\left(\begin{matrix}b_1& b_2& b_3& \cdots& b_n\\0& b_2& b_3& \cdots& b_n\\\vdots&\ddots& b_3&\cdots&b_n\\\vdots& & \ddots& \ddots& \vdots\\0& \cdots& \cdots& 0& b_n\\\end{matrix}\right)\left(\begin{matrix}b_1&0& \cdots& \cdots& 0\\b_2& b_2& \ddots& & \vdots\\b_3& b_3& b_3& \ddots& \vdots\\\vdots& \vdots& \vdots& \ddots&0\\b_n& b_n& b_n& \cdots& b_n\\\end{matrix}\right)=B^TB,\]
其中
\[\left\{ \begin{array}{l}b_1^2 + b_2^2 + b_3^2 +  \cdots  + b_n^2 = 1,\\b_2^2 + b_3^2 +  \cdots  + b_n^2 = \frac{1}{2},\\\vdots \\b_n^2 = \frac{1}{n}.\end{array} \right.\]
解得$b_k=\frac1{\sqrt{k(k+1)}},k=1,2,\cdots,n-1$且$b_n=\frac1{\sqrt{n}}$.
 
由Rayleigh商定理可知只需证明
\[0\leq\frac{x^TB^TBx}{x^Tx}=\frac{(Bx)^TBx}{x^Tx}\leq 3+2\sqrt{2},\]
其中$x=(x_1,x_2,\cdots,x_n)^T$.等价于证明
\[0\leq\sum_{k=1}^{n-1}{\frac{\left(x_1+x_2+\cdots +x_k\right)^2}{k\left(k+1\right)}}+\frac{\left(x_1+x_2+\cdots +x_n\right)^2}{n}\leq (3+2\sqrt{2})\sum_{k=1}^n{x_{k}^{2}}.\]
 
左边的不等式是显然的.下面证明右边不等式.事实上,由Cauchy-Schwarz不等式可知
\[\left(\frac{x_{1}^{2}}{a_1}+\frac{x_{2}^{2}}{a_2}+\cdots +\frac{x_{k}^{2}}{a_k}\right)\left(a_1+a_2+\cdots +a_k\right)\geq\left(x_1+x_2+\cdots +x_k\right)^2.\]
不等式可以改写为
\[\frac{\left(x_1+x_2+\cdots +x_k\right)^2}{k\left(k+1\right)}\leq\sum_{i=1}^k{\frac{a_1+a_2+\cdots +a_k}{k\left(k+1\right)a_i}x_{i}^{2}}.\]
对于$k=1,2,\cdots,n-1$,构造类似的不等式,累加得
\[\sum_{k=1}^{n-1}{\frac{\left(x_1+x_2+\cdots +x_k\right)^2}{k\left(k+1\right)}}+\frac{\left(x_1+x_2+\cdots+x_n\right)^2}{n}\leq\sum_{k=1}^n{y_kx_{k}^{2}},\]
其中
\[y_k=\sum_{i=k}^{n-1}{\frac{a_1+a_2+\cdots +a_i}{i\left(i+1\right)a_k}}+\frac{a_1+a_2+\cdots +a_n}{na_k}.\]
只需证明数列$(a_1,a_2,\cdots,a_n)$满足$y_k\leq 3+2\sqrt{2}$即可.我们取$a_k=\sqrt{k}-\sqrt{k-1}$,则$a_1+a_2+\cdots+a_k=\sqrt{k}$.此时,我们有
\[y_k=\frac{1}{a_k}\left(\sum_{i=k}^{n-1}{\frac{1}{\left(i+1\right)\sqrt{i}}}+\frac{1}{\sqrt{n}}\right).\]
注意到
\begin{align*}2\left(\frac{1}{\sqrt{i}}-\frac{1}{\sqrt{i+1}}\right)&=2\frac{\sqrt{i+1}-\sqrt{i}}{\sqrt{i}\cdot\sqrt{i+1}}=2\frac{1}{\sqrt{i}\cdot\sqrt{i+1}\left(\sqrt{i+1}+\sqrt{i}\right)}\\&\geq 2\frac{1}{\sqrt{i}\cdot\sqrt{i+1}\left(\sqrt{i+1}+\sqrt{i+1}\right)}=\frac{1}{\left(i+1\right)\sqrt{i}}.\end{align*}
因此
\begin{align*}y_k&\leq\frac{1}{a_k}\left[\sum_{i=k}^{n-1}{2\left(\frac{1}{\sqrt{i}}-\frac{1}{\sqrt{i+1}}\right)}+\frac{1}{\sqrt{n}}\right]=\frac{1}{a_k}\left[2\left(\frac{1}{\sqrt{k}}-\frac{1}{\sqrt{n}}\right)+\frac{1}{\sqrt{n}}\right]\\&\leq\frac{2}{a_k\sqrt{k}}=\frac{2}{\left(\sqrt{k}-\sqrt{k-1}\right)\sqrt{k}}=\frac{2\left(\sqrt{k}+\sqrt{k-1}\right)}{\sqrt{k}}\\&=2\left(1+\sqrt{1-\frac{1}{k}}\right)<4<3+2\sqrt{2}.\end{align*}

2.(高等概率论)Let the stochastic processes $\{X_k,1\leq k\leq n\}$ and $\{X'_k,1\leq k\leq n\}$ be independent of one another and have the same joint distributions. If $m_k$ is a median of $X_k,1\leq k\leq n$. Prove: for $\lambda>0$,

$$P\left( {\mathop {\max }\limits_{1 \le k \le n} \left| {{X_k} - {m_k}} \right| \ge \lambda } \right) \le 2P\left( {\mathop {\max }\limits_{1 \le k \le n} \left| {{X_k} - {X'_k}} \right| \ge \lambda } \right).$$

证.(DH)令$S_{k}^{\ast}=\underset{1\leq j\leq k}{\max}\left| X_j-m_j\right|$且$S_{0}^{\ast}=0$.记\[A_1=\left\{S_{1}^{\ast}\geq\lambda\right\},\quad A_k=\left\{S_{k-1}^{\ast}<\lambda ,\left| X_k-m_k\right|\geq\lambda\right\},\]

则有$A_1\cup\cdots\cup A_n=\left\{\underset{1\leq k\leq n}{\max}\left| X_k-m_k\right|\geq\lambda\right\}$且$A_i\cap A_j=\emptyset,i\neq j$.并记

\[A_k^+=\left\{S_{k-1}^{\ast}<\lambda , X_k-m_k\geq\lambda\right\},\quad A_k=\left\{S_{k-1}^{\ast}<\lambda , X_k-m_k\leq -\lambda\right\},\]则有$A_k^+\cap A_k^-=\emptyset,A_k^+\cup A_k^-=A_k$,且$\forall 1\leq i<j\leq n, A_i^+\cap A_j^+=A_i^-\cap A_j^-=\emptyset$.又因为$A_i\cap A_j=\emptyset$,则$\forall 1\leq i<j\leq n,A_i^+\cap A_j^-=\emptyset$.再令$M_{k}^{+}=\left\{m_k-X'_k\geq 0\right\},M_{k}^{-}=\left\{m_k-X'_k\leq 0\right\}$且\[B_k=\left\{\underset{1\leq j\leq k-1}{\max}\left| X_j-X'_j\right|<\lambda ,\left| X_k-X'_k\right|\geq\lambda\right\},\]

\[P\left(\underset{1\leq k\leq n}{\max}\left| X_k-X'_k\right|\geq\lambda\right)=P\left(\bigcap_{k=1}^n{B_k}\right).\]

仿照$A_k^+,A_k^-$,构造$B_k^+,B_k^-$,亦有$B_i^+\cap B_j^-=\emptyset,\forall 1\leq i,j \leq n$成立.因此

\[P\left(\underset{1\leq k\leq n}{\max}\left| X_k-X'_k\right|\geq\lambda\right)=P\left(\bigcap_{k=1}^n{B_k^+}\right)+P\left(\bigcap_{k=1}^n{B_k^-}\right).\]

 

因为$X_k,X'_k$同分布,所以$m_k=m'_k$且$X_k-m_k+m_k-X'_k=0$,所以$B_k^+\supset A_k^+\cap M_k^+,B_k^-\supset A_k^-\cap M_k^-$.又因为$\left\{X_k\right\}$与$\left\{X'_k\right\}$独立,因此

\begin{align*}P\left(\underset{1\leq k\leq n}{\max}\left| X_k-X'_k\right|\geq\lambda\right)&\geq P\left(\bigcap_{k=1}^n{\left(A_{k}^{+}\cap M_{k}^{+}\right)}\right)+P\left(\bigcap_{k=1}^n{\left(A_{k}^{-}\cap M_{k}^{-}\right)}\right)\\&=\sum_{k=1}^n{P\left(A_{k}^{+}\right)\cdot P\left(M_{k}^{+}\right)}+\sum_{k=1}^n{P\left(A_{k}^{-}\right)\cdot P\left(M_{k}^{-}\right)}\\&=\frac{1}{2}\sum_{k=1}^n{\left[P\left(A_{k}^{+}\right)+P\left(A_{k}^{-}\right)\right]}=\frac{1}{2}P\left(\bigcap_{k=1}^n{\left(A_{k}^{+}\cap A_{k}^{-}\right)}\right)\\&=\frac{1}{2}P\left(\bigcap_{k=1}^n{A_k}\right)=\frac{1}{2}P\left\{\underset{1\leq k\leq n}{\max}\left| X_k-m_k\right|\geq\lambda\right\}.\end{align*}

 
3.(高等代数)$F$为数域, $A,B,P\in M_n(F)$, $P$幂零且\[(A-B)P=P(A-B),\qquad BP-PB=2(A-B).\]

求一个可逆矩阵$Q$使得$AQ=QB$.


证.(SUCCEME)记$H=A-B,G=P/2$,则我们有$HP=PH,BG-GB=H$,只需取一个可逆矩阵$Q$使得$(B+H)Q=QB$.

事实上,我们可取\[Q = {e^{ - G}} = 1 - G + \frac{{{G^2}}}{{2!}} - \frac{{{G^3}}}{{3!}} + \cdots \]

由$P$幂零可知上面的和为有限和.由$e^G\cdot e^{-G}=I$可知$Q$可逆.又由

\begin{align*}B{G^n} - {G^n}B&= \left( {B{G^n} - GB{G^{n - 1}}} \right) + \left( {GB{G^{n - 1}} - {G^2}B{G^{n - 1}}} \right) + \cdots + \left( {{G^{n - 1}}BG - {G^n}B} \right)\\&= H{G^{n - 1}} + GH{G^{n - 2}} + \cdots + {G^{n - 1}}H = nH{G^{n - 1}},\end{align*}

可知

\[B{G^n} - {G^n}B=nH{G^{n - 1}}.\]

因此

\begin{align*}\left( {B + H} \right)Q - QB &= \left( {B + H} \right)\left( {1 - G + \frac{{{G^2}}}{{2!}} - \frac{{{G^3}}}{{3!}} + \cdots } \right) - \left( {1 - G + \frac{{{G^2}}}{{2!}} - \frac{{{G^3}}}{{3!}} + \cdots } \right)B\\&= \sum\limits_{n \ge 0} {\left[ {\left( {B + H} \right)\frac{{{{\left( { - 1} \right)}^n}{G^n}}}{{n!}} - \frac{{{{\left( { - 1} \right)}^n}{G^n}}}{{n!}}B} \right]} = \sum\limits_{n \ge 0} {\frac{{{{\left( { - 1} \right)}^n}}}{{n!}}\left( {B{G^n} - {G^n}B + H{G^n}} \right)} \\&= \sum\limits_{n \ge 0} {\frac{{{{\left( { - 1} \right)}^n}}}{{n!}}\left( {nH{G^{n - 1}} + H{G^n}} \right)} = \sum\limits_{n \ge 0} {{{\left( { - 1} \right)}^n}\left( {\frac{{H{G^{n - 1}}}}{{\left( {n - 1} \right)!}} + \frac{{H{G^n}}}{{n!}}} \right)} = 0.\end{align*}

这里约定$n=-1$时, $HP^n/n!=0$.所以此时$\left( {B + H} \right)Q = QB$成立.

曾经的两道矩阵正定问题的解答

设$A\in M_n (\mathbb{R})$对称正定, $X\in M_{n\times m} (\mathbb{R})$且$X^TX=I_m$.证明$X^TA^{-1}X-{X^TAX}^{-1}$是半正定矩阵.


证.首先

\[\left( {\begin{array}{*{20}{c}}{{X^T}AX}&{{I_m}}\\{{I_m}}&{{X^T}{A^{ - 1}}X}\end{array}} \right) = \left( {\begin{array}{*{20}{c}}{{X^T}AX}&{{X^T}X}\\{{X^T}X}&{{X^T}{A^{ - 1}}X}\end{array}} \right) = \left( {\begin{array}{*{20}{c}}{{X^T}}&0\\0&{{X^T}}\end{array}} \right)\left( {\begin{array}{*{20}{c}}A&{{I_n}}\\{{I_n}}&{{A^{ - 1}}}\end{array}} \right)\left( {\begin{array}{*{20}{c}}X&0\\0&X\end{array}} \right).\]

\[\left( {\begin{array}{*{20}{c}}{{I_n}}&0\\{ - {A^{ - 1}}}&{{I_n}}\end{array}} \right)\left( {\begin{array}{*{20}{c}}A&{{I_n}}\\{{I_n}}&{{A^{ - 1}}}\end{array}} \right)\left( {\begin{array}{*{20}{c}}{{I_n}}&{ - {A^{ - 1}}}\\0&{{I_n}}\end{array}} \right) = \left( {\begin{array}{*{20}{c}}A&0\\0&0\end{array}} \right).\]

所以$\left( {\begin{array}{*{20}{c}}A&{{I_n}}\\{{I_n}}&{{A^{ - 1}}}\end{array}} \right)$半正定,所以$\left( {\begin{array}{*{20}{c}}{{X^T}AX}&{{I_m}}\\{{I_m}}&{{X^T}{A^{ - 1}}X}\end{array}} \right)$半正定.

 

又\[\left( {\begin{array}{*{20}{c}}{{I_m}}&0\\{ - {{\left( {{X^T}AX} \right)}^{ - 1}}}&{{I_m}}\end{array}} \right)\left( {\begin{array}{*{20}{c}}{{X^T}AX}&{{I_m}}\\{{I_m}}&{{X^T}{A^{ - 1}}X}\end{array}} \right)\left( {\begin{array}{*{20}{c}}{{I_m}}&{ - {{\left( {{X^T}AX} \right)}^{ - 1}}}\\0&{{I_m}}\end{array}} \right) = \left( {\begin{array}{*{20}{c}}{{X^T}AX}&0\\0&{{X^T}{A^{ - 1}}X - {{\left( {{X^T}AX} \right)}^{ - 1}}}\end{array}} \right),\]

所以$X^TA^{-1}X-{X^TAX}^{-1}$是半正定矩阵.


设 $x_i>0,i=1,2,\cdots$, 证明矩阵 $${\left( {\frac{{\ln \left( {1 + {x_i} + {x_j}} \right) - \ln \left( {1 + \left| {{x_i} - {x_j}} \right|} \right)}}{{{x_i} + {x_j} - \left| {{x_i} - {x_j}} \right|}}} \right)_{n \times n}} $$是半正定矩阵.


证.(morrismodel)这是一道合成题, 我把它分解开来:

 

题1. 当$0<x_1<x_2<\cdots<x_n$时,

$$\left(\min\{x_i,x_j\}\right)_{n\times n}= \begin{pmatrix} x_1&x_1&x_1&\cdots&x_1\\ x_1&x_2&x_2&\cdots &x_2\\ x_1&x_2&x_3&\cdots&x_3\\ \cdots&\cdots&\cdots&\cdots\\ x_1&x_2&x_3&\cdots&x_n \end{pmatrix} $$

正定.

 

证明. 由如下的矩阵恒等式得到:

\begin{align*}&\begin{pmatrix} a_1&\\ a_1&a_2\\ a_1&a_2&a_3\\ \cdots&\cdots&\cdots&\cdots\\ a_1&a_2&a_3&\cdots&a_n \end{pmatrix} \begin{pmatrix} a_1&a_1&a_1&\cdots&a_1\\ 0&a_2&a_2&\cdots &a_2\\ 0&0&a_3&\cdots&a_3\\ \cdots&\cdots&\cdots&\cdots\\ 0&0&0&\cdots&a_n \end{pmatrix}\\=& \begin{pmatrix} a_1^2&a_1^2&a_1^2&\cdots&a_1^2\\ a_1^2&a_1^2+a_2^2&a_1^2+a_2^2&\cdots &a_1^2+a_2^2\\ a_1^2&a_1^2+a_2^2&a_1^2+a_2^2+a_3^2&\cdots&a_1^2+a_2^2+a_3^2\\ \cdots&\cdots&\cdots&\cdots\\ a_1^2&a_1^2+a_2^2&a_1^2+a_2^2+a_3^2&\cdots&a_1^2+a_2^2+a_3^2+\cdots+a_n^2 \end{pmatrix}.\end{align*}

 

题2. 当正数$x_1,x_2,\cdots,x_n$两两不相等时,

$$\left(\min\{x_i,x_j\}\right)_{n\times n}= \begin{pmatrix} x_1&x_1&x_1&\cdots&x_1\\ x_1&x_2&x_2&\cdots &x_2\\ x_1&x_2&x_3&\cdots&x_3\\ \cdots&\cdots&\cdots&\cdots\\ x_1&x_2&x_3&\cdots&x_n \end{pmatrix} $$

正定.

 

证明. 对任何$N$阶置换$\sigma\in S_N$,

\begin{align*} &\sum_{i,j=1}^Na_{i}a_{j}\min\{x_{\sigma(i)},x_{\sigma(j)}\}=\sum_{i=1}^N\left(\sum_{j=1}^Na_{i}a_{j}\min\{x_{\sigma(i)},x_{\sigma(j)}\}\right)\\=&\sum_{i=1}^N\left(\sum_{j=1}^Na_{i}a_{\sigma^{-1}(j)}\min\{x_{\sigma(i)},x_{j}\}\right)=\sum_{j=1}^N\left(\sum_{i=1}^Na_{i}a_{\sigma^{-1}(j)}\min\{x_{\sigma(i)},x_{j}\}\right)\\=&\sum_{j=1}^N\left(\sum_{i=1}^Na_{\sigma^{-1}(i)}a_{\sigma^{-1}(j)}\min\{x_{i},x_{j}\}\right)=\sum_{i,j=1}^Na_{\sigma^{-1}(i)}a_{\sigma^{-1}(j)}\min\{x_{i},x_{j}\}. \end{align*}

再结合题1就得到结论.

 

题3. 当正数$x_1,x_2,\cdots,x_n$两两不相等时,

$$\left(e^{x_i+x_j-|x_i-x_j|}\right)_{n\times n}= \begin{pmatrix} e^{2x_1}&e^{2x_1}&e^{2x_1}&\cdots&e^{2x_1}\\ e^{2x_1}&e^{2x_2}&e^{2x_2}&\cdots &e^{2x_2}\\ e^{2x_1}&e^{2x_2}&e^{2x_3}&\cdots&e^{2x_3}\\ \cdots&\cdots&\cdots&\cdots\\ e^{2x_1}&e^{2x_2}&e^{2x_3}&\cdots&e^{2x_n} \end{pmatrix} $$

正定.

 

证明. 由题2得到.

 

题4. 当正数$x_1,x_2,\cdots,x_n$两两不相等且$\theta\in[0,1]$时,

$$\left(\frac{1}{1+\theta(x_i+x_j)+(1-\theta)|x_i-x_j|}\right)_{n\times n} $$

正定.

 

证明.

\begin{align*} &\sum_{i,j=1}^N\frac{a_ia_j}{1+\theta(x_i+x_j)+(1-\theta)|x_i-x_j|}\\=&\sum_{i,j=1}^Na_ia_j\int_0^\infty e^{-t(1+\theta(x_i+x_j)+(1-\theta)|x_i-x_j|)}dt\\ =&\int_0^\infty e^{-t}\left(\sum_{i,j=1}^N(a_ie^{-tx_i})(a_je^{-tx_j})e^{t(1-\theta)(x_i+x_j-|x_i-x_j|)}\right)dt. \end{align*}

再由题3得到结论.

 

 

题5. 当正数$x_1,x_2,\cdots,x_n$两两不相等时,

$$\left( {\frac{{\ln \left( {1 + {x_i} + {x_j}} \right) - \ln \left( {1 + \left| {{x_i} - {x_j}} \right|} \right)}}{{{x_i} + {x_j} - \left| {{x_i} - {x_j}} \right|}}} \right)_{n \times n} $$

正定.

 

证明.

$${\frac{{\ln \left( {1 + {x_i} + {x_j}} \right) - \ln \left( {1 + \left| {{x_i} - {x_j}} \right|} \right)}}{{{x_i} + {x_j} - \left| {{x_i} - {x_j}}\right|}}}=\int_0^1 \frac{1}{1+\theta(x_i+x_j)+(1-\theta)|x_i-x_j|} d\theta.$$

再由题4得到结论.

 

回到原题, 没有假设正数$x_i$两两不等, 只需在题5的基础上摄动一下就能得到半正定性.


 

2011年南开大学高等代数试题

 

参考:http://www.math.org.cn/forum.php?mod=viewthread&tid=21850&extra=&page=2

许以超书上一行列式求解

许以超第二版书上P73页留了个行列式求解的思考题,还是比较棘手的,下面给出自己的解答.


\[{\Delta _n} = \det \left( {\begin{array}{*{20}{c}}{1 + {x_1}{y_1}}&{1 + {x_1}{y_2}}& \cdots &{1 + {x_1}{y_n}}\\{1 + {x_1}y_1^2}&{1 + {x_1}y_2^2}& \cdots &{1 + {x_1}y_n^2}\\\vdots & \vdots &{}& \vdots \\{1 + {x_1}y_1^n}&{1 + {x_1}y_2^n}& \cdots &{1 + {x_1}y_n^n}\end{array}} \right).\]

解:先来个引理(许以超自己给出的).


事实上,我们有

\begin{align*}&\det A + x\sum\limits_{j,k = 1}^n {{A_{jk}}} = \det \left( {\begin{array}{*{20}{c}}{{a_{11}} + x}& \cdots &{{a_{1n}} + x}\\\vdots &{}& \vdots \\{{a_{n1}} + x}& \cdots &{{a_{nn}} + x}\end{array}} \right)\\=& \det A + x\det \left( {\begin{array}{*{20}{c}}1&1& \cdots &1&1\\{{a_{21}} - {a_{11}}}&{{a_{22}} - {a_{12}}}& \cdots &{{a_{2,n - 1}} - {a_{1,n - 1}}}&{{a_{2n}} - {a_{1n}}}\\{{a_{31}} - {a_{21}}}&{{a_{32}} - {a_{22}}}& \cdots &{{a_{3,n - 1}} - {a_{2,n - 1}}}&{{a_{3n}} - {a_{2n}}}\\\vdots & \vdots &{}& \vdots & \vdots \\{{a_{n1}} - {a_{n - 1,1}}}&{{a_{n2}} - {a_{n - 1,2}}}& \cdots &{{a_{n,n - 1}} - {a_{n - 1,n - 1}}}&{{a_{nn}} - {a_{n - 1,n}}}\end{array}} \right).\end{align*}

其中$A_{kj}$是方阵$A=(a_{jk})$的第$k$行,第$j$列位置的元素的代数余子式.


因此

\begin{align*}&{\Delta _n} = \det \left( {\begin{array}{*{20}{c}}{1 + {x_1}{y_1}}&{1 + {x_1}{y_2}}& \cdots &{1 + {x_1}{y_n}}\\{1 + {x_1}y_1^2}&{1 + {x_1}y_2^2}& \cdots &{1 + {x_1}y_n^2}\\\vdots & \vdots &{}& \vdots \\{1 + {x_1}y_1^n}&{1 + {x_1}y_2^n}& \cdots &{1 + {x_1}y_n^n}\end{array}} \right)\\= &\det A + \det \left( {\begin{array}{*{20}{c}}1&1& \cdots &1\\{{x_1}\left( {y_1^2 - {y_1}} \right)}&{{x_1}\left( {y_2^2 - {y_2}} \right)}& \cdots &{x_1\left( {y_n^2 - {y_n}} \right)}\\\vdots & \vdots &{}& \vdots \\{{x_1}\left( {y_1^n - y_1^{n - 1}} \right)}&{{x_1}\left( {y_2^n - y_2^{n - 1}} \right)}& \cdots &{{x_1}\left( {y_n^n - y_n^{n - 1}} \right)}\end{array}} \right)\\=& \det A + x_1^{n - 1}\det \left( {\begin{array}{*{20}{c}}1&1& \cdots &1\\{y_1^2 - {y_1}}&{y_2^2 - {y_2}}& \cdots &{y_n^2 - {y_n}}\\\vdots & \vdots &{}& \vdots \\{y_1^n - y_1^{n - 1}}&{y_2^n - y_2^{n - 1}}& \cdots &{y_n^n - y_n^{n - 1}}\end{array}} \right).\end{align*}

 

对上面的行列式进行升阶:

\[\det \left( {\begin{array}{*{20}{c}}1&1& \cdots &1\\{y_1^2 - {y_1}}&{y_2^2 - {y_2}}& \cdots &{y_n^2 - {y_n}}\\\vdots & \vdots &{}& \vdots \\{y_1^n - y_1^{n - 1}}&{y_2^n - y_2^{n - 1}}& \cdots &{y_n^n - y_n^{n - 1}}\end{array}} \right) = \det \left( {\begin{array}{*{20}{c}}1&{{y_1}}&{{y_2}}& \cdots &{{y_n}}\\0&1&1& \cdots &1\\0&{y_1^2 - {y_1}}&{y_2^2 - {y_2}}& \cdots &{y_n^2 - {y_n}}\\\vdots & \vdots & \vdots &{}& \vdots \\0&{y_1^n - y_1^{n - 1}}&{y_2^n - y_2^{n - 1}}& \cdots &{y_n^n - y_n^{n - 1}}\end{array}} \right).\]

将第一行加到第三行,第三行加到第四行,$\cdots$,最后将第$n$行加到第$n+1$行:

\begin{align*}\det \left( {\begin{array}{*{20}{c}}1&{{y_1}}&{{y_2}}& \cdots &{{y_n}}\\0&1&1& \cdots &1\\1&{y_1^2}&{y_2^2}& \cdots &{y_n^2}\\\vdots & \vdots & \vdots &{}& \vdots \\1&{y_1^n}&{y_2^n}& \cdots &{y_n^n}\end{array}} \right) &= \det \left( {\begin{array}{*{20}{c}}1&{{y_1}}&{{y_2}}& \cdots &{{y_n}}\\1&1&1& \cdots &1\\1&{y_1^2}&{y_2^2}& \cdots &{y_n^2}\\\vdots & \vdots & \vdots &{}& \vdots \\1&{y_1^n}&{y_2^n}& \cdots &{y_n^n}\end{array}} \right) + \det \left( {\begin{array}{*{20}{c}}1&{{y_1}}&{{y_2}}& \cdots &{{y_n}}\\{ - 1}&0&0& \cdots &0\\1&{y_1^2}&{y_2^2}& \cdots &{y_n^2}\\\vdots & \vdots & \vdots &{}& \vdots \\1&{y_1^n}&{y_2^n}& \cdots &{y_n^n}\end{array}} \right)\\& = \left( { - 1} \right) \cdot \prod\limits_{k = 1}^n {\left( {{y_k} - 1} \right)} \cdot \prod\limits_{1 \le i < j \le n} {\left( {{y_j} - {y_i}} \right)} + \prod\limits_{k = 1}^n {{y_k}} \cdot \prod\limits_{1 \le i < j \le n} {\left( {{y_j} - {y_i}} \right)} \\& = \prod\limits_{1 \le i < j \le n} {\left( {{y_j} - {y_i}} \right)} \cdot \left[ { - \prod\limits_{k = 1}^n {\left( {{y_k} - 1} \right)} + \prod\limits_{k = 1}^n {{y_k}} } \right].\end{align*}

因此

\begin{align*}{\Delta _n} &= \det A + x_1^{n - 1}\prod\limits_{1 \le i < j \le n} {\left( {{y_j} - {y_i}} \right)} \cdot \left[ { - \prod\limits_{k = 1}^n {\left( {{y_k} - 1} \right)} + \prod\limits_{k = 1}^n {{y_k}} } \right]\\& = x_1^n \cdot \prod\limits_{k = 1}^n {{y_k}} \cdot \prod\limits_{1 \le i < j \le n} {\left( {{y_j} - {y_i}} \right)} + x_1^{n - 1}\prod\limits_{1 \le i < j \le n} {\left( {{y_j} - {y_i}} \right)} \cdot \left[ { - \prod\limits_{k = 1}^n {\left( {{y_k} - 1} \right)} + \prod\limits_{k = 1}^n {{y_k}} } \right]\\& = x_1^{n - 1}\prod\limits_{1 \le i < j \le n} {\left( {{y_j} - {y_i}} \right)} \cdot \left[ { - \prod\limits_{k = 1}^n {\left( {{y_k} - 1} \right)} + \left( {{x_1} + 1} \right)\prod\limits_{k = 1}^n {{y_k}} } \right].\end{align*}

一个求特征值的问题

今天13级数院的JJ发来提问:


求对称矩阵

\[A = \left( {\begin{array}{*{20}{c}}{a_{11}^2}&{{a_{11}}{a_{12}} + 1}&{{a_{11}}{a_{13}} + 1}& \cdots &{{a_{11}}{a_{1n}} + 1}\\{{a_{11}}{a_{12}} + 1}&{a_{22}^2}&{{a_{22}}{a_{23}} + 1}& \cdots &{{a_{22}}{a_{2n}} + 1}\\{{a_{11}}{a_{13}} + 1}&{{a_{22}}{a_{23}} + 1}& \ddots & \ddots & \vdots \\\vdots & \vdots & \ddots &{a_{n - 1,n - 1}^2}&{{a_{n - 1,n - 1}}{a_{n - 1,n}} + 1}\\{{a_{11}}{a_{1n}} + 1}&{{a_{22}}{a_{2n}} + 1}& \cdots &{{a_{n - 1,n - 1}}{a_{n - 1,n}} + 1}&{a_{nn}^2}\end{array}} \right)\]的特征值.

2015年武汉大学高等代数考研试题

源自:http://www.math.org.cn/forum.php?mod=viewthread&tid=31895&highlight=%E6%AD%A6%E6%B1%89%E5%A4%A7%E5%AD%A6

浙江大学2012年研究生入学考试高等代数试题参考解答

浙江大学2012年研究生入学考试高等代数试题参考解答
 
 
高等代数资源网
 
2013.8.2
1. 声明
您现在看到的这份文件来自http://www.52gd.org.本站原创的内容,采用创作共用组织(Creative Commons)的“公共领域”(\href{http://creativecommons.org/about/pdm}{Public Domain})许可。即放弃一切权利,全归公共领域。但涉及到其他版权人的摘录、转载、投稿、翻译等类内容不在此列。
 
本文的内容仅供学习参考之用,作者不对内容的正确性作任何承诺,作者不对因使用本文而造成的一切后果承担任何责任.
 
关于如何使用本文的建议:首先保证自己认真做了一遍题目,否则请不要查看本文.记住:
 
别人做是别人的,自己做才是自己的 .
作者水平有限,错误不可避免,欢迎您来信指出:\href{mailto:www52gdorg@163.com}{www52gdorg@163.com}.
 
2. 试题

每题15分.

 

一.设$E$是$n$阶单位矩阵,

$$M=\begin{pmatrix}0&E\\-E&0\end{pmatrix},$$

矩阵$A$满足$A^{T}MA=M.$证明$A$的行列式等于1.

 

二.设$A$是$n$阶幂等矩阵,满足

 

(1)$A=A_{1}+\cdots+A_{s};$

 

(2)$r(A)=r(A_{1})+\cdots+r(A_{s}).$

 

证明:所有的$A_{i}$都相似于一个对角阵,$A_{i}$的特征值之和等于矩阵$A_{i}$的秩.

 

三.设$\phi$是$n$维欧氏空间的正交变换,证明:$\phi$最多可以表示为$n+1$个镜面反射的复合.

 

四.设$A$是$n$阶复矩阵,证明存在常数项等于0的多项式$g(\lambda),h(\lambda)$使得$g(A)$是可以对角化的矩阵,$h(A)$是幂零矩阵,且$A=g(A)+h(A).$

 

五.设$A=\begin{pmatrix}3&2&-2\\k&-1&-k\\4&2&-3\end{pmatrix}.$(i)当$k$为何值时,存在矩阵$P$使得$P^{-1}AP$为对角矩阵?并求出这样的矩阵$P$和对角矩阵.(ii)求$k=2$时矩阵$A$的Jordan标准形.

 

六.令二次型$f(x_{1},\cdots,x_{n})=\sum_{i=1}^{m}(a_{i1}x_{1}+\cdots+a_{in}x_{n})^{2}.$

 

(i)求此二次型的方阵.

 

(ii)当$a_{ij}$均为实数时,给出此二次型为正定的条件.

 

七.设$V$和$W$是数域$K$上的线性空间,$Hom_{K}(V,W)$表示$V$到$W$的所有线性映射组成的线性空间.证明:对$f,g\in Hom_{K}(V,W),$若$Imf\cap Img=\{0\},$则$f,g$在$Hom_{K}(V,W)$中是线性无关的.

 

八.令线性空间$V=Imf\oplus W,$其中$W$是线性变换$f$的不变子空间.

 

(i)证明$W\subseteq Kerf;$

 

(ii)证明若$V$是有限维线性空间,则$W=Kerf;$

 

(iii)举例说明,当$V$是无限维的,可能有$W\subseteq Ker f,$且$W\neq Kerf.$

 

九.设$A=\begin{pmatrix}1&0&-1&2&1\\-1&1&3&-1&0\\-2&1&4&-1&3\\3&-1&-5&1&-6\end{pmatrix}.$

 

(i)求$5\times 5$阶秩为2的矩阵$M,$使得$AM=0;$

 

(ii)假如$B$是满足$AB=0$的$5\times5$阶矩阵,证明:秩$\mathrm{rank\,}(B)\leq2.$

 

十.令$T$是有限维线性空间$V$的线性变换,设$W$是$V$的$T-$不变子空间.那么$T|_{W}$的最小多项式整除$T$的最小多项式.

 

 

 

3. 参考解答

一.设$E$是$n$阶单位矩阵,

$$M=\begin{pmatrix}0&E\\-E&0\end{pmatrix},$$

矩阵$A$满足$A^{T}MA=M.$证明$A$的行列式等于1.

 

\textbf{证明:}(法1)将$A$分块为

$$A=\begin{pmatrix}A_{1}&A_{2}\\A_{3}&A_{4}\end{pmatrix}$$

由$A^{T}MA=M$有

$$\begin{aligned}-A_{2}A_{1}^{T}+A_{1}A_{2}^{T}&=&0\\-A_{2}A_{3}^{T}+A_{1}A_{4}^{T}&=&I\\\end{aligned}$$

 

若$A_{1}$可逆,由$A$的分块可得$$|A|=|A_{1}||A_{4}-A_{3}A_{1}^{-1}A_{2}|$$由上面第一式可得$$A_{2}=A_{1}A_{2}^{T}(A_{1}^{-1})^{T}$$

代入第二式可得

$$A_{4}-A_{3}A_{1}^{-1}A_{2}=(A_{1}^{-1})^{T}$$

从而可得$|A|=1$.

 

当$A_{1}$不可逆时,考虑矩阵$A_{1}+tE,$则存在无穷多$t$的值使得$A_{1}+tE$可逆(这是因为$|A_{1}+tE|$是关于$t$的一个多项式,只能有有限个根.),由前面的证明有

$$1=\begin{vmatrix}A_{1}+tE&A_{2}\\A_{3}&A_{4}\end{vmatrix}.$$

此式两边是关于$t$的多项式,且有无穷多$t$的值使得等式成立,从而等式恒成立.令$t=0$可得.

 

(法2)博士数学论坛(\url{www.math.org.cn})的mxcandy提供.

 

由$A^{T}MA=M$有

$$M(\lambda E-A)=\lambda M-MA=\lambda A^{T}MA-MA=(\lambda A^{T}-E)MA.$$

两边取行列式,由$|M|=1\neq0$得

$$|\lambda E-A|=|A||\lambda A^{T}-E|.$$

注意到$A$是$2n$阶矩阵,以及矩阵的转置行列式不变有

$$|\lambda A^{T}-E|=|E-\lambda A^{T}|=|(E-\lambda A^{T})^{T}|=|E-\lambda A|.$$

于是

$$|\lambda E-A|=|A||\lambda A^{T}-E|=|A||E-\lambda A|.$$

记$A$的特征多项式$|\lambda E-A|=f(\lambda),$则由上式有

$$f(\lambda)=|\lambda E-A|=|A|\lambda^{2n}f(\dfrac{1}{\lambda}).\eqno(*)$$

考虑$\lambda=1,$设$2n$次多项式$f(\lambda)$有分解式

$$f(\lambda)=(\lambda-1)^{m}g(\lambda),g(1)\neq0,0\leq m\leq 2n.$$

已知$A^{T}MA=M,$两边取行列式,可得$|A|^{2}=1,$从而$A$可逆,故

$$MA=(A^{T})^{-1}M,$$

由平滑性或者归纳法可得,对任意自然数$k$有

$$M(E-A)^{k}=(E-(A^{T})^{-1})^{k}M,$$

从而

$$M(E-A)^{2k}=M(E-A)^{k}(E-A)^{k}=(E-(A^{T})^{-1})^{k}M(E-A)^{k}=-(A^{T})^{-k}(E-A)^{k}M(E-A)^{k}.$$

由$M^{T}=-M$知,$(E-A)^{k}M(E-A)^{k}$反对称,注意到$|M|=1,$从而

$$\mathrm{rank\,}((E-A)^{2k})=\mathrm{rank\,}((E-A)^{k}M(E-A)^{k}).$$

由于反对称矩阵的秩为偶数,从而$\mathrm{rank\,}((E-A)^{2k})$为偶数.特别的,任取$2k\geq m,$则特征值$\lambda=1$的代数重数$m$为偶数,即

$$m=2n-\mathrm{rank\,}((E-A)^{2k})\triangleq 2p,2k\geq m,0\leq p\leq n.$$

把$f(\lambda)=(\lambda-1)^{2p}g(\lambda)$代入到(*)式,得

$$(\lambda-1)^{2p}g(\lambda)=|A|\lambda^{2n}(\dfrac{1}{\lambda}-1)^{2p}g(\dfrac{1}{\lambda}),$$

$$g(\lambda)=|A|\lambda^{2n-2k}g(\dfrac{1}{\lambda}),$$

令$\lambda=1,$并注意到$g(1)\neq0,$可得$|A|=1.$

 

注1:满足题目条件的矩阵$A$称为辛矩阵.

 

注2:由上述证明知:辛矩阵的特征多项式自反,特征值互倒成对,$\lambda=\pm1$代数重数为偶数.

 

(法3)许以超,线性代数与矩阵论(第二版).高等教育出版社.P329.

 

(法4)许以超,线性代数与矩阵论(第二版).高等教育出版社.P405.

 

(法5)高等代数中的一些问题.博士数学论坛(\url{www.math.org.cn})xida.P7.

二.设$A$是$n$阶幂等矩阵,满足

 

(1)$A=A_{1}+\cdots+A_{s};$

 

(2)$r(A)=r(A_{1})+\cdots+r(A_{s}).$

 

证明:所有的$A_{i}$都相似于一个对角阵,$A_{i}$的特征值之和等于矩阵$A_{i}$的秩.

 

\textbf{证明:}只需证明$A_{i}$是幂等矩阵.利用$n$阶矩阵$C$是幂等矩阵的充要条件为$r(C)+r(C-E)=n,$只需证明$r(A_{i}-E)=n-r(A_{i}).$利用矩阵秩的不等式

$$|r(A)-r(B)|\leq r(A\pm B)\leq r(A)+r(B)$$

以及题目条件有

$$\begin{aligned}n-r(A_{i})\leq r(A_{i}-E)&=r(A-E-(A_{1}+\cdots+A_{i-1}+A_{i+1}+\cdots+A_{s}))\\&\leq r(A-E)+r(A_{1}+\cdots+A_{i-1}+A_{i+1}+\cdots+A_{s})\\&\leq r(A-E)+r(A_{1})+\cdots+r(A_{i-1})+r(A_{i+1})+\cdots+r(A_{s})\\&=n-r(A)+r(A)-r(A_{i})\\&=n-r(A_{i}).\end{aligned}$$

从而$r(A_{i}-E)=n-r(A_{i}).$

 

三.设$\phi$是$n$维欧氏空间的正交变换,证明:$\phi$最多可以表示为$n+1$个镜面反射的复合.

 

\textbf{证明:}(法1)设$\alpha$是$n$维欧氏空间$V$中的单位向量,定义线性变换

$$\sigma(\beta)=\beta-(\beta,\alpha)\alpha,\forall\beta\in V,$$

则$\sigma$是$V$的正交变换,称为镜面反射(镜像变换).计算可得$\sigma^{2}=I$(恒等变换).

 

设$\alpha_{1},\alpha_{2}$是$n$维欧氏空间$V$中的两个长度相等的不同向量,则存在镜面反射$\sigma$使得$\sigma(\alpha_{1})=\alpha_{2}.$

实际上,令$\alpha=\dfrac{\alpha_{1}-\alpha_{2}}{|\alpha_{1}-\alpha_{2}|},$定义$\sigma(\beta)=\beta-(\beta,\alpha)\alpha,\forall\beta\in V$即可.

 

下面证明原问题.对空间的维数$n$用数学归纳法.

 

当$n=1$时,设$e_{1}$是$V$的单位向量,则$V=L(e_{1}).$由于$\phi(e_{1})\in V,$故存在实数$\lambda$使得$\phi(e_{1})=\lambda e_{1},$由$\phi$是正交变换可得

$$1=(e_{1},e_{1})=(\phi(e_{1}),\phi(e_{1}))=\lambda^{2}(e_{1},e_{1})=\lambda^{2},$$

因此$\lambda=\pm1.$令

$$\tau(\alpha)=\alpha-2(\alpha,e_{1})e_{1},\forall\alpha\in V,$$

则$\tau$是镜面反射,且当$\lambda=1$时,对$\forall\alpha\in V,$设$\alpha=ke_{1},k\in R$则

$$\phi(\alpha)=k\phi(e_{1})=ke_{1},\forall\alpha=ke_{1}=\alpha,\in V,$$

即$\phi$是恒等变换.而

$$\tau^{2}(\alpha)=k\tau^{2}(e_{1})=k\tau(-e_{1})=ke_{1}=\alpha,$$

即$\tau^{2}$也是恒等变换,从而$\phi=\tau^{2}.$而当$\lambda=-1$时,显然$\phi=\tau.$

 

假设结论对$n-1$维欧氏空间成立,对$n$维欧氏空间$V$的一正交变换$\phi,$若$\phi=I,$则对$V$的任一镜面反射$\sigma$有$\phi=I=\sigma^{2}.$若$\phi\neq I,$则存在$V$的单位向量$e$使得$\phi(e)=\eta\neq e,$由于$|\eta|=|\phi(e)|,$从而存在$V$的镜面反射$\tau$使得$$\tau(\eta)=e.$$

于是$$\tau(\phi(e))=e.$$

令$W=L(e),$由于$\tau\phi$仍为正交变换,故$W^{\bot}$是$\tau\phi$的$n-1$维不变子空间,且$\tau\phi|_{W^{\bot}}$为正交变换.由归纳假设,在$W^{\bot}$中存在单位向量$\alpha_{1},\alpha_{2},\cdots,\alpha_{k},$它们分别决定$W^{\bot}$的镜面反射$\sigma_{1},\sigma_{2},\cdots,\sigma_{k}$使得

$$\tau\phi|_{W^{\bot}}=\sigma_{1}\sigma_{2}\cdots\sigma_{k},$$

现将$\sigma_{i}$的定义扩大到$V,$即补充定义$\sigma_{i}(e)=e.$则$\sigma_{i}$即为$\alpha_{i}$决定的$V$的镜面反射.这是因为$\forall\alpha\in V,$设$\alpha=\beta_{1}+\beta_{2},\beta_{1}=ke\in W=L(e),\beta_{2}\in W^{\bot},$注意到$(\beta_{1},\alpha_{i})=0,$则

 

$$\begin{aligned}\sigma_{i}(\alpha)&=\sigma_{i}(\beta_{1})+\sigma(\beta_{2})\\&=\beta_{1}+\beta_{2}-2(\beta_{2},\alpha_{i})\alpha_{i}\\&=\alpha-2(\alpha,\alpha_{i})\alpha_{i}.\end{aligned}$$

现在显然有$\sigma_{1}\sigma_{2}\cdots\sigma_{k}(\beta_{1})=\beta_{1},$这是因为$\tau\phi(e)=e,$故$\tau\phi(\beta_{1})=\beta_{1}.$从而

 

$$\begin{aligned}\tau\phi(\alpha)&=\tau\phi(\beta_{1})+\tau\phi(\beta_{2})\\&=\beta_{1}+\sigma_{1}\sigma_{2}\cdots\sigma_{k}(\beta_{2})\\&\sigma_{1}\sigma_{2}\cdots\sigma_{k}(\beta_{1})+\sigma_{1}\sigma_{2}\cdots\sigma_{k}(\beta_{2})\\&=\sigma_{1}\sigma_{2}\cdots\sigma_{k}(\alpha)\end{aligned}$$

 

从而$\tau\phi=\sigma_{1}\sigma_{2}\cdots\sigma_{k}.$注意到$\tau^{2}=I,$有

$$\phi=\tau\sigma_{1}\sigma_{2}\cdots\sigma_{k}.$$

 

(法2)$n$阶矩阵$M=E-2\alpha\alpha^{T},$其中$\alpha$是$n$维实列向量,且$\alpha^{T}\alpha=1.$则矩阵$M$是正交矩阵,称为镜像矩阵.容易验证$M^{2}=E.$即单位矩阵是两个镜像矩阵之积.

 

设$\alpha,\beta$是两个不同的$n$维实列向量,且$|\alpha|=|\beta|,$则存在实镜像矩阵$M$使得$M\alpha=\beta.$实际上,令$\alpha=\dfrac{\alpha-\beta}{|\alpha-\beta|},M=E-2\alpha\alpha^{T}$即可.

 

可以证明欧氏空间中的线性变换$\phi$是镜面反射的充要条件是$\phi$在一组标准正交基下的矩阵为镜像矩阵.

 

这样要证明原问题,只需证明任意$n$阶实正交矩阵$A$可以分解不超过$n+1$个镜像矩阵之积即可.

 

对矩阵的阶数$n$用数学归纳法.

$n=1$时,结论显然成立.

 

假设结论对$n-1$阶矩阵成立,将$n$阶正交矩阵$A$按列分块为

$$A=(\alpha_{1},\alpha_{2},\cdots,\alpha_{n}),$$

则$|\alpha_{1}|=1,$从而存在镜像矩阵$M_{1}$使得$M_{1}\alpha_{1}=(1,0,\cdots,0)^{T},$注意到$M_{1}A$还是正交矩阵,必有

 

 

$$M_{1}A=M_{1}(\alpha_{1},\alpha_{2},\cdots,\alpha_{n})=(M_{1}\alpha_{1},M_{1}\alpha_{2},\cdots,M_{1}\alpha_{n})=\begin{pmatrix}1&0&\cdots&0\\0& &  &\\\vdots&&Q_{1}&\\0&&&\end{pmatrix}$$

 

 

容易验证$Q_{1}$也是正交矩阵,从而由归纳假设,存在$n-1$阶镜像矩阵$M_{2},\cdots,M_{k}$使得$$Q_{1}=M_{2}\cdots M_{k},$$

于是

$$A=M_{1}^{-1}\begin{pmatrix}1&0&\cdots&0\\0& &  &\\\vdots&&Q_{1}&\\0&&&\end{pmatrix}=M_{1}^{-1}\begin{pmatrix}1&\\&M_{2}\cdots M_{k}\end{pmatrix}=M_{1}^{-1}\begin{pmatrix}1&\\&M_{2}\end{pmatrix}\cdots\begin{pmatrix}1&\\&M_{k}\end{pmatrix}.$$

 

易知$\begin{pmatrix}1&\\&M_{i}\end{pmatrix}$都是镜像矩阵.

 

 

 

四.设$A$是$n$阶复矩阵,证明存在常数项等于0的多项式$g(\lambda),h(\lambda)$使得$g(A)$是可以对角化的矩阵,$h(A)$是幂零矩阵,且$A=g(A)+h(A).$

 

\textbf{证明:}等我看看能否找到一个好的方法.

 

五.设$A=\begin{pmatrix}3&2&-2\\k&-1&-k\\4&2&-3\end{pmatrix}.$(i)当$k$为何值时,存在矩阵$P$使得$P^{-1}AP$为对角矩阵?并求出这样的矩阵$P$和对角矩阵.(ii)求$k=2$时矩阵$A$的Jordan标准形.

 

\textbf{证明:}由于

$$|A-\lambda E|=\begin{vmatrix}3-\lambda&2&-2\\k&-1-\lambda&-k\\4&2&-3-\lambda\end{vmatrix}=-(\lambda+1)^{2}(\lambda-1),$$

故$A$的特征值为$$\lambda_{1}=-1(\mbox{二重}),\lambda_{2}=1.$$

 

(i)存在矩阵$P$使得$P^{-1}AP$为对角矩阵的充要条件是特征值的代数重数等于几何重数,即$r(A-\lambda_{1}E)=1,$而

$$A-\lambda_{1}E=\begin{pmatrix}4&2&-2\\k&0&-k\\4&2&-2\end{pmatrix},$$

 

从而$k=0.$

$P$可以是

$$P=\begin{pmatrix}1&1&1\\-2&0&0\\0&2&1\\\end{pmatrix},$$此时$P^{-1}AP=diag(-1,-1,1).$

 

(2)$k=2$时

 

$$\lambda E-A=\begin{pmatrix}\lambda-3&-2&2\\-2&\lambda+1&2\\-4&-2&\lambda+3\end{pmatrix}\rightarrow\begin{pmatrix}1&&\\&1&\\&&(\lambda+1)^{2}(\lambda-1)\end{pmatrix},$$

 

所以$A$的Jordan标准形为$$\begin{pmatrix}-1&1&\\&-1&\\&&1\end{pmatrix}.$$

 

六.令二次型$f(x_{1},\cdots,x_{n})=\sum_{i=1}^{m}(a_{i1}x_{1}+\cdots+a_{in}x_{n})^{2}.$

 

(i)求此二次型的方阵.

 

(ii)当$a_{ij}$均为实数时,给出此二次型为正定的条件.

 

\textbf{证明:}(i)由于

$$(a_{i1}x_{1}+\cdots+a_{in}x_{n})^{2}=(x_{1},\cdots,x_{n})\begin{pmatrix}a_{i1}\\\vdots\\a_{in}\end{pmatrix}\begin{pmatrix}a_{i1}&\cdots&a_{in}\end{pmatrix}\begin{pmatrix}x_{1}\\\vdots\\x_{n}\end{pmatrix},$$

 

\begin{align*}f(x_{1},\cdots,x_{n})&=\sum_{i=1}^{m}(a_{i1}x_{1}+\cdots+a_{in}x_{n})^{2}\\&=\sum_{i=1}^{n}(x_{1},\cdots,x_{n})\begin{pmatrix}a_{i1}\\\vdots\\a_{in}\end{pmatrix}\begin{pmatrix}a_{i1}&\cdots&a_{in}\end{pmatrix}\begin{pmatrix}x_{1}\\\vdots\\x_{n}\end{pmatrix}\\&=(x_{1},\cdots,x_{n})\begin{pmatrix}\sum_{i=1}^{n}a_{i1}^{2}&\cdots&\sum_{i=1}^{n}a_{i1}a_{in}\\\vdots&\vdots&\vdots\\\sum_{i=1}^{n}a_{in}a_{i1}&\cdots&\sum_{i=1}^{n}a_{in}^{2}\end{pmatrix}\begin{pmatrix}x_{1}\\\vdots\\x_{n}\end{pmatrix}\end{align*}.

 

若记$A=(a_{ij})_{n\times n},$则$f(x_{1},\cdots,x_{n})=(x_{1},\cdots,x_{n})(A^{T}A)\begin{pmatrix}x_{1}\\\vdots\\x_{n}\end{pmatrix}.$

 

故所求矩阵为$A^{T}A.$

 

(2)当$a_{ij}$为实数时,$A^{T}A$是半正定的,故$f(x_{1},\cdots,x_{n})=(x_{1},\cdots,x_{n})(A^{T}A)\begin{pmatrix}x_{1}\\\vdots\\x_{n}\end{pmatrix}$

正定的充要条件是$r(A^{T}A)=n.$而$r(A^{T}A)=r(A),$故原二次型正定的充要条件是$r(A)=n.$

 

七.设$V$和$W$是数域$K$上的线性空间,$Hom_{K}(V,W)$表示$V$到$W$的所有线性映射组成的线性空间.证明:对$f,g\in Hom_{K}(V,W),$若$Imf\cap Img=\{0\},$则$f,g$在$Hom_{K}(V,W)$中是线性无关的.

 

\textbf{证明:}注:这里应该假设$f\neq0,g\neq0.$否则题目无意义.

反证法.假设$f=kg,k\in K,$由于$f\neq0,$故存在$\alpha\in V,$使得$0\neq f(\alpha)\in Im f\subset W,$此时

$$o\neq \dfrac{1}{k}f(\alpha)=g(\alpha)\in Img,$$

注意到$Img$是$W$的字空间,从而$f(\alpha)\in Img,$这样$0\neq f(\alpha)\in Imf\cap Img.$这与条件矛盾.

 

八.令线性空间$V=Imf\oplus W,$其中$W$是线性变换$f$的不变子空间.

 

(i)证明$W\subseteq Kerf;$

 

(ii)证明若$V$是有限维线性空间,则$W=Kerf;$

 

(iii)举例说明,当$V$是无限维的,可能有$W\subseteq Ker f,$且$W\neq Kerf.$

 

\textbf{证明:}(i)$\forall\alpha\in W,$则由条件有

$$f(\alpha)\in Imf\cap W,$$

注意到$V=Imf\oplus W,$从而$Imf\cap W=\{0\},$故$f(\alpha)=0.$即$\alpha\in Kerf.$这就证明了$W\subseteq Kerf.$

 

(2)由(i),要证明$W=Kerf,$只需证明$dim W=dim Kerf.$而由$V=Imf\oplus W$以及维数公式$dimV=dim Imf+dim Kerf$有

$$dim W=dimV-dim Imf=dim Kerf.$$

从而结论成立.

 

(3)例:$V=P[x]$是数域$P$上关于$x$的一元多项式的全体,则$V$是无限维线性空间,$f(p(x))=p'(x)$为$V$上的求导线性变换,则

此时$Imf=V,Kerf=P,W=\{0\}.$

 

九.设$A=\begin{pmatrix}1&0&-1&2&1\\-1&1&3&-1&0\\-2&1&4&-1&3\\3&-1&-5&1&-6\end{pmatrix}.$

 

(i)求$5\times 5$阶秩为2的矩阵$M,$使得$AM=0;$

 

(ii)假如$B$是满足$AB=0$的$5\times5$阶矩阵,证明:秩$\mathrm{rank\,}(B)\leq2.$

 

\textbf{证明:}将$M$按列分块为

$$M=(m_{1},m_{2},m_{3},m_{4},m_{5}),$$

$$0=AM=A(m_{1},m_{2},m_{3},m_{4},m_{5})=(Am_{1},Am_{2},Am_{3},Am_{4},Am_{5}),$$

即$Am_{i}=0,i=1,2,3,4,5.$此即$m_{i}$是线性方程组$Ax=0$的解.

 

(i)求解$Ax=0$可得其一个基础解系为

$$\alpha_{1}=(-1,2,1,0)^{T},\alpha_{2}=(3,1,0,-2,0)^{T}.$$

故可取

$$M=(\alpha_{1},\alpha_{2},\alpha_{1},\alpha_{1},\alpha_{1}).$$

 

(ii)注意到$B$的列向量是方程组$Ax=0$的解,而方程组的任一解皆可由其基础解系线性表示,故$B$的列向量可由$\alpha_{1},\alpha_{2}$线性表示,故$r(B)\leq2.$

十.令$T$是有限维线性空间$V$的线性变换,设$W$是$V$的$T-$不变子空间.那么$T|_{W}$的最小多项式整除$T$的最小多项式.

 

\textbf{证明:}易知$W$是平凡子空间,即$W=\{0\}\mbox{或}W=V$时,结论成立.

 

下面假设$0<dimW=r<dimV=n,$取$W$的一组基$\alpha_{1},\cdots,\alpha_{r},$将其扩充为$V$的一组基$\alpha_{1},\cdots,\alpha_{r},\alpha_{r+1},\cdots,\alpha_{n},$由$W$是$T$的不变子空间,则可知$T$在上述基下的矩阵为

$$T(\alpha_{1},\cdots,\alpha_{r},\alpha_{r+1},\cdots,\alpha_{n})=(\alpha_{1},\cdots,\alpha_{r},\alpha_{r+1},\cdots,\alpha_{n})\begin{pmatrix}A_{r\times r}&B\\0&C_{(n-r)\times(n-r)}\end{pmatrix}.$$

设$T|_{W},T$的最小多项式分别为$m_{T}(x),m(x),$则

$$0=m(\begin{pmatrix}A_{r\times r}&B\\0&C_{(n-r)\times(n-r)}\end{pmatrix})=\begin{pmatrix}m(A_{r\times r})&*\\0&m(C_{(n-r)\times(n-r)})\end{pmatrix},$$

从而$m(A)=0,$即$m(x)$是$T|_{W}$的零化多项式,从而$m_{T}(x)|m(x).$

转载自:http://www.52gd.org/?p=414